You play a game that requires rolling a six- sided dice then randomly choosing a colored card from a deck containing 3 cards, 8 blue cards, and 6 yellow cards. Find the probability that you will roll a 2 on the dice and then choose a blue card

Answers

Answer 1

Answer:

4/51

Step-by-step explanation:

The probability of rolling a 2 is 1/6.

There are a total of 17 cards, so the probability of a blue card is 8/17.

The probability of both events is:

(1/6) (8/17) = 4/51

Answer 2

We want to find the probability of rolling a 2 and then getting a blue card. We will see that this probability is equal to 0.078

First, we must find the probability of rolling a 2 with a six-sided dice.

A six-sided dice has 6 possible outcomes, only one of these is a 2, then the probability of rolling a 2 is just:

p = 1/6

Then we find the probability of drawing a blue card.

There are:

3 cards (no color)8 blue cards6 yellow cards.

So there is a total of 3 + 8 + 6 = 17 cards, and 8 of these are blue, then the probability of randomly drawing a blue card is:

q = 8/17

The joint probability is given by the product of the two individual probabilities, so we get:

P = p*q = (1/6)*(8/17) = 0.078

So the probability of rolling a 2 and then getting a blue card is P = 0.078

If you want to learn more about probability, you can read:

https://brainly.com/question/17223269


Related Questions

Students are designing a new town as part of a social studies project on urban planning. ey want to place the town’s high school at point A and the middle school at point B. they also plan to build roads that run directly from point A to the mall and from point B to the mall. the average cost to build a road in this area is $550,000 per mile. a. Find the measure of each acute angle of the right triangle shown. b. Find the length of the hypotenuse. Also nd the length of each of the three congruent segments forming the hypotenuse.

Answers

Answer:

The downtown angle measures about 22.62° and the town pool angle measures about 67.38°.

The hypotenuse measures 13 miles. Each third measures 4 1/3 miles.

Step-by-step explanation:

Use trigonometric ratios to solve this problem.

a. Find the measure of each acute angle of the right triangle shown.

Let's start with the acute angle that's marked near downtown. We can use the trigonometric ratio tangent to find the measure of this angle. (Remember, tangent = opposite/adjacent!)

We can divide the opposite side from the angle by the adjacent side to find the tangent:

5/12

= 0.4166666...

Now, we do the inverse tangent to find the measure of the angle:

≈ 22.62°

Now, we can find the angle near the town pool. This time, we can use tangent again, but the 12 mi side is the opposite and the 5 mi side is the adjacent:

12/5

= 2.4

Now, calculate the inverse tangent:

≈ 67.38°

b. Find the length of the hypotenuse. Also find the length of each of the three congruent segments forming the hypotenuse.

We can use the Pythagorean theorem to find the length of the hypotenuse. Remember, given legs a and b and hypotenuse c, the Pythagorean theorem states:

a² + b² = c²

Plug the values in this triangle into this equation:

5² + 12² = c²

25 + 144 = c²

169 = c²

13 = c

The hypotenuse measures 13 miles.

Now, we find the length of the three congruent segments that form the hypotenuse. (to be honest, I'm not sure that there are three congruent segments, but oh well, I'll just go with what it says there).

Since all the segments are the same length, we can just divide 13 by 3 to find the length of each of them:

13/3 = 4 1/3.

Each of the segments measures 4 1/3 miles.

A paper company produces 4,675 notebooks in 5 days. How many notebooks can it produce in 13 days?
OA. 12,155
OB. 2,431
OC. 4,473
OD. 12,100

Answers

Answer:

OA. 12,155

Step-by-step explanation:

A paper company produces 4,675 notebooks in 5 days. How many notebooks can it produce in 13 days?

--------

in 5 days= 4675

in 13 days= 4675/5*13= 12155

Answer

A OR D

Step-by-step explanation:

A bag of trail mix shrugged 1.625 pounds round 1.625 to the nearest tenth.use the number line for help

Answers

Answer:

1.625 when convert it to 1.6 pounds

Step-by-step explanation:

The given bag of mill shrugged 1. 625 pounds.

1.625 ≈ 1.6 poundest to the nearest tenth.

The federal government recently granted funds for a special program designed to reduce crime in high-crime areas. A study of the results of the program in eight high-crime areas of Miami, Florida, yielded the following results.
Number of Crimes by Area
A B C D E F G H
before 14 7 4 5 17 12 8 9
after 2 7 3 6 8 13 3 5
Has there been a decrease in the number of crimes since the inauguration of the program? Use the .01 significance level. Estimate the p-value

Answers

Answer:

Step-by-step explanation:

Hello!

There was a special program funded, designed to reduce crime in 8 areas of Miami.

The number of crimes per area was recorded before and after the program was established in each area. This is an example of a paired data situation. For each are in Miami you have recorded a pair of values:

X₁: Number of crimes recorded in one of the eight areas of Miami before applying the special program.

X₂: Number of crimes recorded in one of the eight areas of Miami after applying the special program.

Area: (Before; After)

A: (14; 2)

B: (7; 7)

C; (4; 3)

D: (5; 6)

E: (17; 8)

F: (12; 13)

G: (8; 3)

H; (9; 5)

To apply a paired sample test you have to define the variable "difference":

Xd= X₁ - X₂

I'll define it as the difference between the crime rate before the program and after the program.

If the original populations have a normal distribution, we can assume that the  variable defined from them will also have a normal distribution.

Xd~N(μd; σd²)

If the crime rate decreased after the special program started, you'd expect the population mean of the difference between the crime rates before and after the program started to be less than zero, symbolically μd<0

The hypotheses are:

H₀: μd≥0

H₁: μd<0

α: 0.01

[tex]t= \frac{X[bar]_d-Mu_d}{\frac{S_d}{\sqrt{n} } } ~~t_{n-1}[/tex]

To calculate the sample mean and standard deviation of the variable difference, you have to calculate the difference between each value of each pair:

A= 14 - 2= 12

B= 7 - 7= 0

C= 4 - 3= 1

D= 5 - 6= -1

E= 17 - 8= 9

F= 12 - 13= -1

G= 8 - 3= 5

H= 9 - 5= 4

∑Xdi= 12 + 0 + 1 + (-1) + 9 + (-1) + 5 + 4= 29

∑Xdi²= 12²+0²+1²+1²+9²+1²+5²4²= 269

X[bar]d= 29/8= 3.625= 3.63

[tex]S_d=\sqrt{\frac{1}{n-1}[sumX_d^2-\frac{(sumX_d)^2}{n} ] } = \sqrt{\frac{1}{7}[269-\frac{29^2}{8} ] } = 4.84[/tex]

[tex]t_{H_0}= \frac{3.63-0}{\frac{4.86}{\sqrt{8} } } = 2.11[/tex]

This test is one-tailed to the left and so is the p-value, under a t with n-1= 8-1=7 degrees of freedom, the probability of obtaining a value as extreme as the calculated value is:

P(t₇≤-2.11)= 0.0364

The p-value is greater than the significance level, so the decision is to not reject the null hypothesis. Then at a 1% significance level, you can conclude that the special program didn't reduce the crime rate in the 8 designated areas of Miami.

I hope it helps!

Using the t-distribution, it is found that since the p-value of the test is 0.048 > 0.01, there is not enough evidence to conclude that there has been a decrease in the number of crimes since the inauguration of the program.

At the null hypothesis, it is tested if there has been no reduction, that is, the subtraction of the mean after by the mean before is at least 0, hence:

[tex]H_0: \mu_A - \mu_B \geq 0[/tex]

At the alternative hypothesis, it is tested if there has been a reduction, that is, the subtraction of the mean after by the mean before is negative, hence:

[tex]H_1: \mu_A - \mu_B < 0[/tex]

For both before and after, the mean, standard deviation of the sample(this is why the t-distribution is used) and sample sizes are given by:

[tex]\mu_B = 9.5, s_B = 4.504, n_B = 8[/tex]

[tex]\mu_A = 5.875, s_A = 3.5632, n_A = 8[/tex]

The standard errors are given by:

[tex]s_A = \frac{3.5632}{\sqrt{8}} = 1.2596[/tex]

[tex]s_B = \frac{4.504}{\sqrt{8}} = 1.5924[/tex]

For the distribution of differences, the mean and standard error are given by:

[tex]\overline{x} = \mu_A - \mu_B = 5.875 - 9.5 = -3.625[/tex]

[tex]s = \sqrt{s_A^2 + s_B^2} = \sqrt{1.2596^2 + 1.5924^2} = 2.0304[/tex]

The test statistic is given by:

[tex]t = \frac{\overline{x} - \mu}{s}[/tex]

In which [tex]\mu = 0[/tex] is the value tested at the null hypothesis.

Hence:

[tex]t = \frac{\overline{x} - \mu}{s}[/tex]

[tex]t = \frac{-3.625 - 0}{2.0304}[/tex]

[tex]t = -1.7854[/tex]

The p-value is found using a t-distribution calculator, with t = -1.7854, 8 + 8 - 2 = 14 df and a left-tailed test with a significance level of 0.01, as we are tested if the mean is less than a value.

Using the calculator, the p-value is given by 0.048.

Since the p-value of the test is 0.048 > 0.01, there is not enough evidence to conclude that there has been a decrease in the number of crimes since the inauguration of the program.

You can learn more about the use of the t-distribution to test an hypothesis at https://brainly.com/question/13873630

A man claims that his lot is triangular, with one side 450 m long and the adjacent side 200 m long. The


angle opposite one side is 28º. Determine the other side length of this lot to the nearest metre.

Answers

Answer:

Accurate answer: 617 mAnswer out of available options: C. 616 m

Step-by-step explanation:

Given information:

Side a = 450 m (opposite angle A)Side b = 200 m (opposite angle B)Angle A = 28°

We can use the Law of Sines to find angle B:

[tex]$\frac{a}{\sin{A}} = \frac{b}{\sin{B}}[/tex]

Substitute the given values:

[tex]$\frac{450}{\sin{28^\circ}} = \frac{200}{\sin{B}}[/tex]

Now, solve for angle B:

[tex]$\sin{B} = \frac{200 \times \sin{28^\circ}}{450}[/tex][tex]$\sin{B} \approx 0.208654[/tex][tex]$B \approx \arcsin{0.208654} \approx 12.0435^\circ[/tex]

Now that we have angle B, we can find angle C using the fact that the sum of the interior angles in a triangle is always 180°:

Angle C = 180° - 28° - 12.0435° Angle C = 139.9565°

Now, we can use the Law of Sines again to find the length of the other side (side c) opposite angle C:

[tex]$\frac{a}{\sin{A}} = \frac{c}{\sin{C}}[/tex]

Substitute the given values:

[tex]$\frac{450}{\sin{28^\circ}} = \frac{c}{\sin{139.9565^\circ}}[/tex]

Now, solve for side c:

[tex]$c = \frac{450 \times \sin{139.9565^\circ}}{\sin{28^\circ}}[/tex][tex]$c \approx 616.685[/tex]

To the nearest meter, the other side length of the triangular lot is approximately 617 m. But, since there is not an option for the answer, the closest option is C. 616 m.

________________________________________________________

Full Question

Answer:

  289 m  or  617 m

Step-by-step explanation:

You want the third side length of a triangle with side lengths 450 m and 200 m, with an angle of 28°.

Solution 1

The man's claim does not say which side the given angle is opposite. There are two possibilities. (1) It is opposite the unknown side; (2) it is opposite the side of length 450 m. (No triangle is possible having an angle of 28° opposite the shorter given side.)

If the angle is opposite the unknown side, the law of cosines can be used to find the third side length:

  c² = a² + b² - 2ab·cos(C)

  c² = 450² +200² -2·450·200·cos(28°) ≈ 83569.43

  c ≈ √83569.43 ≈ 289 . . . . meters

The other side length could be 289 meters.

Solution 2

The third side could also be figured using the law of sines.

  a/sin(A) = b/sin(B) = c/sin(C)

  450/sin(28°) = 200/sin(B)

  B = arcsin(200/450·sin(28°)) ≈ 12.043°

Then angle C is ...

  C = 180° -28° -12.043° = 139.957°

and side 'c' is ...

  c = 450·sin(139.957°)/sin(28°) ≈ 617 . . . . meters

The other side length could be 617 meters.

__

Additional comment

The problem tells us "one side" is 450 m, and it tells us the angle opposite "one side" is 28°. If both of the descriptors "one side" are referring to the same side, then Solution 2 is the intended one.

The description can be written in a less ambiguous way. As is, we are not sure that the second use of "one side" is referring to any side in particular. Hence the two possibilities.

<95141404393>

An engineering school reports that 52% of its students are male (M), 33% of its students are between the ages of 18 and 20 (A), and that 27% are both male and between the ages of 18 and 20. What is the probability of a random student being chosen who is a female and is not between the ages of 18 and 20?

Answers

Answer:

42%

Step-by-step explanation:

Given: P(M) = 0.52, P(A) = 0.33, and P(M and A) = 0.27.

Find: P(not M and not A).

P(not M and not A) = 1 − P(M or A)

P(not M and not A) = 1 − (P(M) + P(A) − P(M and A))

P(not M and not A) = 1 − (0.52 + 0.33 − 0.27)

P(not M and not A) = 1 − 0.58

P(not M and not A) = 0.42

Treating these probabilities as Venn probabilities, it is found that there is a 0.42 = 42% probability of a random student being chosen who is a female and is not between the ages of 18 and 20.

-------------------------

The events are:

Event A: Female.Event B: Not between the ages of 18 and 20.

-------------------------

52% of the students are male, thus, 48% are female, and [tex]P(A) = 0.48[/tex].33% are between the ages of 18 and 20, thus, 67% are not between these ages, which means that [tex]P(B) = 0.67[/tex]27% are both male and between these ages, which means that 73% are either female or not between these ages, thus [tex]P(A \cup B) = 0.73[/tex].

-------------------------

The probability of a random student being chosen who is a female and is not between the ages of 18 and 20 is given by:

[tex]P(A \cap B) = P(A) + P(B) - P(A \cup B)[/tex]

Inserting the probabilities we found:

[tex]P(A \cap B) = 0.48 + 0.67 - 0.73 = 0.42[/tex]

0.42 = 42% probability of a random student being chosen who is a female and is not between the ages of 18 and 20.

A similar problem is given at https://brainly.com/question/21421475

Please answer this correctly

Answers

Answer:

x = 48

Step-by-step explanation:

Since it's given that these two shapes are similar, you can set up a proportion to solve for x, like so:

[tex]\frac{27}{18} =\frac{72}{x}[/tex]

Cross multiply:

[tex]\frac{27x}{1296}[/tex]

Divide 1296 by 27:

x = 48

If the Math Olympiad Club consists of 11 students, how many different teams of 3 students can be formed for competitions?

Answers

Answer:

165 different teams of 3 students can be formed for competitions

Step-by-step explanation:

Combinations of m elements taken from n in n (m≥n) are called all possible groupings that can be made with the m elements so that:

Not all items fitNo matter the order Elements are not repeated

That is, a combination is an arrangement of elements where the place or position they occupy within the arrangement does not matter. In a combination it is interesting to form groups and their content.

To calculate the number of combinations, the following expression is applied:

[tex]C=\frac{m!}{n!*(m-n)!}[/tex]

It indicates the combinations of m objects taken from among n objects, where the term "n!" is called "factorial of n" and is the multiplication of all the numbers that go from "n" to 1.

In this case:

n: 3m: 11

Replacing:

[tex]C=\frac{11!}{3!*(11-3)!}[/tex]

Solving:

[tex]C=\frac{11!}{3!*8!}[/tex]

being:

3!=3*2*1=68!=8*7*6*5*4*3*2*1=40,32011!=39,916,800

So:

[tex]C=\frac{39,916,800}{6*40,320}[/tex]

C= 165

165 different teams of 3 students can be formed for competitions

Answer:

There will be 3 teams of 3 students, and one team of 2 students, so there will be 4 teams with one team one student short, but only 3 teams that can hold 3 students

can anyone please explain me this,would be appreciated

Brian, a scientist, is writing a research paper on projectile motion. during one of his experiments, he throws a ball from a point marked as point a, with a certain velocity in the horizontal direction. the ball travels a horizontal distance of 0.6 meter in the 1st second, 1.2 meters in the 2nd second, 1.8 meters in the 3rd second, and so on. it hits the ground on the 8th second. brian marks the point where the ball landed as point b. calculate the distance between point a and point b.

Answers

Answer:

21.6m

Step-by-step explanation:

Brian throws a ball from point 'a'

The ball travels a distance of:

0.6m in the 1^st second1.2m in the 2^nd second1.8m in the 3^rd second2.4m in the 4^th second3.0m in the 5^th second3.6m in the 6^th second4.2m in the 7^th second4.8m in the 8^th second

The ball travels a total distance of 0.6m + 1.2m + 1.8m + 2.4m + 3.0m + 3.6m + 4.2m + 4.8m = 21.6m from point 'a' to point 'b'.

A walk-in medical clinic believes that arrivals are uniformly distributed over weekdays (Monday through Friday). It has collected the following data based on a random sample of 100 days. Frequency Mon 25 Tue 22 Wed 19 Thu 18 Fri 16 Total 100 Assuming that a goodness-of-fit test is to be conducted using a 0.10 level of significance, the critical value is:

Answers

Answer:

The degrees of freedom are given by;

[tex] df =n-1= 5-1=4[/tex]

The significance level is 0.1 so then the critical value would be given by:

[tex] F_{cric}= 7.779[/tex]

If the calculated value is higher than this value we can reject the null hypothesis that the arrivals are uniformly distributed over weekdays

Step-by-step explanation:

For this case we have the following observed values:

Mon 25 Tue 22 Wed 19 Thu 18 Fri 16 Total 100

For this case the expected values for each day are assumed:

[tex] E_i = \frac{100}{5}= 20[/tex]

The statsitic would be given by:

[tex] \chi^2 = \sum_{i=1}^n \frac{(O_i-E_i)^2}{E_i}[/tex]

Where O represent the observed values and E the expected values

The degrees of freedom are given by;

[tex] df =n-1= 5-1=4[/tex]

The significance level is 0.1 so then the critical value would be given by:

[tex] F_{cric}= 7.779[/tex]

If the calculated value is higher than this value we can reject the null hypothesis that the arrivals are uniformly distributed over weekdays

Assume that the random variable X is normally​ distributed, with mean muequals45 and standard deviation sigmaequals10. Compute the probability ​P(57 > than X less than or = 69​).

Answers

Answer:

0.1069 = 10.69%

Step-by-step explanation:

Problems of normally distributed samples are solved using the z-score formula.

In a set with mean [tex]\mu[/tex] and standard deviation [tex]\sigma[/tex], the zscore of a measure X is given by:

[tex]Z = \frac{X - \mu}{\sigma}[/tex]

The Z-score measures how many standard deviations the measure is from the mean. After finding the Z-score, we look at the z-score table and find the p-value associated with this z-score. This p-value is the probability that the value of the measure is smaller than X, that is, the percentile of X. Subtracting 1 by the pvalue, we get the probability that the value of the measure is greater than X.

In this question:

[tex]\mu = 45, \sigma = 10[/tex]

Between 57 and 69

This is the pvalue of Z when X = 69 subtracted by the pvalue of Z when X = 57. So

X = 69

[tex]Z = \frac{X - \mu}{\sigma}[/tex]

[tex]Z = \frac{69 - 45}{10}[/tex]

[tex]Z = 2.4[/tex]

[tex]Z = 2.4[/tex] has a pvalue of 0.9918

X = 57

[tex]Z = \frac{X - \mu}{\sigma}[/tex]

[tex]Z = \frac{57 - 45}{10}[/tex]

[tex]Z = 1.2[/tex]

[tex]Z = 1.2[/tex] has a pvalue of 0.8849

0.9918 - 0.8849 = 0.1069 = 10.69%

A production line operation is designed to fill cartons with laundry detergent to a mean weight of 32 ounces. A sample of cartons is periodically selected and weighed to determine whether underfilling or overfilling is occurring. If the sample data lead to a conclusion of underfilling or overfilling, the production line will be shut down and adjusted to obtain proper filling. (a) Choose the null and alternative hypotheses that will help in deciding whether to shut down and adjust the production line. H0: - Select your answer - Ha: - Select your answer - (b) Comment on the conclusion and the decision when H0 cannot be rejected. The input in the box below will not be graded, but may be reviewed and considered by your instructor. (c) Comment on the conclusion and the decision when H0 can

Answers

Answer:

See explanation below

Step-by-step explanation:

Given a mean of 32. The claim here is that the mean is 32.

Therefore, the null hypothesis and alternative hypothesis, would be:

H0 : u = 32

Ha: u ≠ 32

b) When we fail to reject null hypothesis, H0. This means that the mean weight, u = 32

Conclusion:  There is not enough evidence to conclude that there is overfilling or underfilling.

c) When null hypothesis, H0 is rejected. This means the mean weight, u ≠ 32.

Conclusion: There is enough evidence to conclude that overfilling or underfilling exists

The null hypothesis in the sampling is u = 32 and the alternative is that u isn't equal to 32.

How is the null hypothesis depicted?

It should be noted that based on the information, when the null hypothesis is rejected, it implies that the weight is 32.

Also, there's no enough evidence to conclude that there's either overfilling or underfilling. When the null hypothesis is rejected, it means that the mean weight is not equal to 32.

Learn more about sampling on:

https://brainly.com/question/17831271

What should the rule be for the table?
On a recent test, you were given the table displayed and
asked to write the rule that models it.
Subtract 6 from the x value to get the y value.
Multiply the x value by 1/2 to get the y value.
Multiply the x value by 1/4 to get the y value.
Add 6 to the x value to get the y value.
8
12
12
6
16
10


(look at picture) pls help

Answers

Answer: it’s A

Step-by-step explanation:

Answer:

I check the answer was a or Subtract 6 from the x value to get the y value

Simplify: 19w5+ (-3075)
Enter the original expression if it cannot be
simplified.
Enter the correct answer.
ODA
DONE

Answers

Simplified: 19w^5 - 3075

A manufacturer produces both a deluxe and a standard model of an automatic sander designed for home use. Selling prices obtained from a sample of retail outlets follow. Model Price ($) Model Price ($) Retail Outlet Deluxe Standard Retail Outlet Deluxe Standard 1 39 27 5 40 30 2 39 29 6 39 35 3 46 35 7 35 29 4 38 31 The manufacturer's suggested retail prices for the two models show a $10 price differential. Use a .05 level of significance and test that the mean difference between the prices of the two models is $10. a.Calculate the value of the test statistic (to 2 decimals).



b.What is the 95% confidence interval for the difference between the mean prices of the two models (to 2 decimals)?

Answers

Answer:

Step-by-step explanation:

The data is incorrect. The correct data is:

Deluxe standard

39 27

39 28

45 35

38 30

40 30

39 34

35 29

Solution:

Deluxe standard difference

39 27 12

39 28 11

45 35 10

38 30 8

40 30 10

39 34 5

35 29 6

a) The mean difference between the selling prices of both models is

xd = (12 + 11 + 10 + 8 + 10 + 5 + 6)/7 = 8.86

Standard deviation = √(summation(x - mean)²/n

n = 7

Summation(x - mean)² = (12 - 8.86)^2 + (11 - 8.86)^2 + (10 - 8.86)^2 + (8 - 8.86)^2 + (10 - 8.86)^2 + (5 - 8.86)^2 + (6 - 8.86)^2 = 40.8572

Standard deviation = √(40.8572/7

sd = 2.42

For the null hypothesis

H0: μd = 10

For the alternative hypothesis

H1: μd ≠ 10

This is a two tailed test.

The distribution is a students t. Therefore, degree of freedom, df = n - 1 = 7 - 1 = 6

2) The formula for determining the test statistic is

t = (xd - μd)/(sd/√n)

t = (8.86 - 10)/(2.42/√7)

t = - 1.25

We would determine the probability value by using the t test calculator.

p = 0.26

Since alpha, 0.05 < than the p value, 0.26, then we would fail to reject the null hypothesis.

b) Confidence interval is expressed as

Mean difference ± margin of error

Mean difference = 8.86

Margin of error = z × s/√n

z is the test score for the 95% confidence level and it is determined from the t distribution table.

df = 7 - 1 = 6

From the table, test score = 2.447

Margin of error = 2.447 × 2.42/√7 = 2.24

Confidence interval is 8.86 ± 2.24

In a given​ year, 94 cities in the world had populations of 1 million or more. Fifty years​ later, 530 cities had populations of 1 million or more. What was the percent​ increase?

Answers

Answer:

The percent increase was of 464%.

Step-by-step explanation:

To find the percent increase, first we find how much the current amount is of the original amount. Then, we subtract the current amount from the original amount.

Percentage of current amount:

We solve this using a rule of three.

The original amount(94 cities), was 100% = 1.

The current amount(530 cities) is x. So

94 cities - 1

530 cities - x

94x = 530

x = 530/94

x = 5.64

5.64 = 564% of the original amount

What was the percent​ increase?

The current amount is 564%

The original amount is 100%

564 - 100 = 464

The percent increase was of 464%.

simplify (3a-2b)²-2(3a-2b)(a+2b)+(a+2)²​

Answers

2. 2
4a - 20ab + 12b. +4a + 4

What’s the correct answer for this?

Answers

9.4 units.

Because,

Formula for arc length is 2times pie times radius times angle divided by 360.

Answer:

The answer is option 2.

Step-by-step explanation:

You have to use length or arc formula, Arc = θ/360×2×π×r where θ represents degrees and r is radius. Then substitute the following values into the formula :

[tex]arc = \frac{θ}{360} \times 2 \times \pi \times r[/tex]

Let θ = 45,

Let r = 12,

[tex]arc = \frac{45}{360} \times 2 \times \pi \times 12[/tex]

[tex]arc = \frac{1}{8} \times 24\pi[/tex]

[tex]arc = 9.42 \: units \: (3s.f)[/tex]

The owner of a senior living facility examines data on the age of the residents at the facility. She finds that the distribution of ages of residents is approximately normal with a mean of 73.5 years and a standard deviation of 6.5 years. Which interval below estimates the middle 99.7% of ages of residents living at the facility?
a. (52,95)
b. (54,93)
c. (60.5, 86,5)
d. (67,80)

Answers

D is the correct answer

The interval of the data if, The mean of 73.5 years and the standard deviation of 6.5 years, is  (67,80) so, option D is correct.

What is mean?

Mean is a measurement of a probability distribution's central tendency along the median and mode. It also goes by the name "anticipated value."

Given:

The mean of 73.5 years and the standard deviation of 6.5 years,

the middle 99.7% of ages of residents living at the facility,

Calculate the interval as shown below,

The coordinates of x in the interval = Mean - Standard deviation

The coordinates of x in the interval = 73.5 - 6.5

The coordinates of x in the interval = 67

The coordinates of y in the interval =  Mean + Standard deviation

The coordinates of y in the interval = 73.5 + 6.5

The coordinates of y in the interval = 80

Thus, the interval will be (67, 80).

To know more about mean:

https://brainly.com/question/2810871

#SPJ5

A study of the amount of time it takes a mechanic to rebuild the transmission for a 1992 Chevrolet Cavalier shows that the mean is 8.4 hours and the standard deviation is 1.8 hours. If 40 mechanics are randomly selected, find the probability that their mean rebuild time exceeds 9.1 hours.
A ) 0.1046 B) 0.0069 C ) 0.1285 D ) 0.0046

Answers

Answer:

B) 0.0069

Step-by-step explanation:

To solve this question, we need to understand the normal probability distribution and the central limit theorem.

Normal probability distribution

Problems of normally distributed samples are solved using the z-score formula.

In a set with mean [tex]\mu[/tex] and standard deviation [tex]\sigma[/tex], the zscore of a measure X is given by:

[tex]Z = \frac{X - \mu}{\sigma}[/tex]

The Z-score measures how many standard deviations the measure is from the mean. After finding the Z-score, we look at the z-score table and find the p-value associated with this z-score. This p-value is the probability that the value of the measure is smaller than X, that is, the percentile of X. Subtracting 1 by the pvalue, we get the probability that the value of the measure is greater than X.

Central Limit Theorem

The Central Limit Theorem estabilishes that, for a normally distributed random variable X, with mean [tex]\mu[/tex] and standard deviation [tex]\sigma[/tex], the sampling distribution of the sample means with size n can be approximated to a normal distribution with mean [tex]\mu[/tex] and standard deviation [tex]s = \frac{\sigma}{\sqrt{n}}[/tex].

For a skewed variable, the Central Limit Theorem can also be applied, as long as n is at least 30.

In this question, we have that:

[tex]\mu = 8.4, \sigma = 1.8, n = 40, s = \frac{1.8}{\sqrt{40}} = 0.2846[/tex]

Find the probability that their mean rebuild time exceeds 9.1 hours.

This is 1 subtracted by the pvalue of Z when X = 9.1. So

[tex]Z = \frac{X - \mu}{\sigma}[/tex]

By the Central Limit Theorem

[tex]Z = \frac{9.1 - 8.4}{0.2846}[/tex]

[tex]Z = 2.46[/tex]

[tex]Z = 2.46[/tex] has a pvalue of 0.9931

1 - 0.9931 = 0.0069

So the answer is B.

If f(x)= 6x squared - 4 and g(x)= 2x + 2 find (f-g)(x)

Answers

Answer:

[tex]6x^2-2x-6[/tex]

Step-by-step explanation:

[tex]f(x)=6x^2-4 \\\\g(x)=2x+2 \\\\(f-g)(x)= (6x^2-4)-(2x+2)=6x^2-2x-6[/tex]

Hope this helps!

find the complete factored form of the polynomial -44a^3 + 20a^6

Answers

Answer:

[tex]4a^3(5a^3-11)[/tex]

Step-by-step explanation:

→Take out the GCF (Greatest Common Factor). The GCF is [tex]4a^3[/tex] because both [tex]-44a^3[/tex] and [tex]20a^6[/tex] can be divided by it, like so:

[tex]-44a^3+20a^6[/tex]

[tex]4a^3(5a^3-11)[/tex]

The temperature at noon was 10 degrees. For the next 3 hours it dropped at a rate of 3 degrees an hour. Express this change in temperature as an integer.

Answers

Answer:

Change in temperature = -9 degrees

Step-by-step explanation:

Given

[tex]Start Temperature = 10 deg[/tex]

[tex]Change = 3 deg/hr[/tex]

[tex]Time= 3hr[/tex]

Required

Change in temperature;

First, the total change in 3 hours has to be solved for. This is done as follows:

[tex]Total Change = Change* Time[/tex]

[tex]Total Change = 3 deg/hr * 3 hr[/tex]

[tex]Total Change = 9 deg[/tex]

From the question, we understand that the temperature dropped.

So,  we can conclude that the temperature changed by -9 degrees;

This means that at the end of the third hour is temperature is 1 degrees

A soft drink machine outputs a mean of 24 ounces per cup. The machine's output is normally distributed with a standard deviation of 3 ounces. What is the probability of filling a cup between 21 and 28 ounces? Round your answer to four decimal places.

Answers

Answer:

[tex]P(21<X<28)=P(\frac{21-\mu}{\sigma}<\frac{X-\mu}{\sigma}<\frac{28-\mu}{\sigma})=P(\frac{21-24}{3}<Z<\frac{28-24}{3})=P(-1<z<1.33)[/tex]

And we can find the probability with this difference

[tex]P(-1<z<1.33)=P(z<1.33)-P(z<-1)[/tex]

And using the normal standard distribution or excel we got:

[tex]P(-1<z<1.33)=P(z<1.33)-P(z<-1)=0.908-0.159=0.749[/tex]

Step-by-step explanation:

Let X the random variable that represent the soft drink machine outputs of a population, and for this case we know the distribution for X is given by:

[tex]X \sim N(24,3)[/tex]  

Where [tex]\mu=24[/tex] and [tex]\sigma=3[/tex]

We want to find this probability:

[tex]P(21<X<28)[/tex]

The z score is given by:

[tex]z=\frac{x-\mu}{\sigma}[/tex]

Using this formula we got:

[tex]P(21<X<28)=P(\frac{21-\mu}{\sigma}<\frac{X-\mu}{\sigma}<\frac{28-\mu}{\sigma})=P(\frac{21-24}{3}<Z<\frac{28-24}{3})=P(-1<z<1.33)[/tex]

And we can find the probability with this difference

[tex]P(-1<z<1.33)=P(z<1.33)-P(z<-1)[/tex]

And using the normal standard distribution or excel we got:

[tex]P(-1<z<1.33)=P(z<1.33)-P(z<-1)=0.908-0.159=0.749[/tex]

Write 7.3 as a mixed number.​

Answers

Answer:

7 3/10

Step-by-step explanation:

7.3=7+0.3= 7+ 3/10= 7 3/10

7 3/10


Place decimal number over a power of ten

Since there is 1 number to the right of the decimal point, place the decimal number over 10^1 (10)

Add whole number to the left of the decimal

Five times the sum of 8u and eight gives one hundred sixty

Answers

Answer:

u=3

Step-by-step explanation:

(8u + 8)5 = 160

40u + 40 = 160

40u = 120

u = 3

Create a word problem on Algebraic Expressions and Measurement (grade 10)​

Answers

Frank can build a fence in twice the time it would take Sandy. Working together, they can build it in 7 hours. How long will it take each of them to do it alone?

Answer

If Frank and Sandy can build the fence in 7 hours, they must be building

1

7

of the fence every hour.

Now, let the amount of time it takes Sandy be

x

hours so that Frank takes

2

x

hours. Sandy can build

1

x

of the fence every hour and Frank can build

1

2

x

of the fence every hour.

We now have the following equation to solve.

1

x

+

1

2

x

=

1

7

2

+

1

2

x

=

1

7

21

=

2

x

x

=

21

2

=

10.50

Thus, Sandy takes

10.50

hours and Frank takes

21

hours.

An account with a $250 balance accrues 2% annually.

Answers

It would increase five dollars in the first year

A manufacturer of large appliances must decide which of two​ machines, A and​ B, they want to purchase to perform a specific task in the production process. The goal is to buy the machine that has smaller mean time required to perform the task. The plant supervisor selects 15 machine operators at​ random, and each operator performs the task on each of the two machines. The production times are paired for each worker. A paired​ t-test is to be performed to determine if there is evidence that the population mean time using machine A is less than the population mean time using machine B. The summary statistics for the differences in the times required for the task in minutes​ (machine A​ - machine​ B) for the 15 randomly selected workers are given below.

n=15; xÌ… = -10.9 and s=20.3

What must be true about the population of differences in the times required for the task between machine A and machine B for conclusions from the paired t-test to be valid for the population of differences among all workers?

a. Because of the small sample size of differences in times required between machine A and machine B, the distribution of sample means of the differences cannot be normal.
b. Because there were a total of 30 obervations (15 times from machine A and 15 times from machine B), the distribution of sample means of the differences will be approximately normal by the Central Limit Theorem.
c. Because the sample size is "large" enough, the distribution of differences for all workers will be normal.
d. Because of the small sample size of differences in times required between machine A and machine B, the distribution of differences for all workers must be normal.

Answers

Answer:

d. Because of the small sample size of differences in times required between machine A and machine B, the distribution of differences for all workers must be normal.

Step-by-step explanation:

A paired t- test conclusion is said to be valid if one of the assumptions that must be satisfied is that: the distribution of the differences must be normal in most cases for which the sample size is small.

From the given information:

the sample size n = 15 ;which is far less than 30

Therefore;we require the distribution of differences in times required between machine A and machine B for all workers to be normal.

From the first option; it is incorrect because even if the sample size is small; the distribution of sample means of the differences will be normal but in the first option ; it is stated that the differences cannot be normal. That makes the first option to be incorrect.

From the second option; is not correct because the sample size (for differences) is 15 and therefore that is a minimal sample which makes the Central Limit Theorem to be invalid and not applicable here.

From the third option; we all know that the sample size is small and not large since it is lesser than 30.

A right triangle has legs 8 and 15. What is its perimeter?
60, 46, 23 or 40?

Answers

Answer:

46

Step-by-step explanation:

Answer:

40

Step-by-step explanation:

8 + 15 + 17 = 40

8 + 15 > 17 15 + 17 > 88 + 17 > 15

**Hope this helped!!**

**If I was right then please mark brainliest!!**

Other Questions
The scene is safe, you've turned on the AED and it prompts you to attach the pads. Choose the best answer for correct both pads placement on a 6 year old child cardiac arrest victim. Clothing Emporium was organized on January 1, 2021. The firm was authorized to issue 140,000 shares of $8 par value common stock. During 2021, Clothing Emporium had the following transactions relating to stockholders' equity: Issued 42,000 shares of common stock at $10 per share. Issued 28,000 shares of common stock at $11 per share. Reported a net income of $140,000. Paid dividends of $70,000. What is total paid-in capital at the end of 2021 find the value of x part 2 What are the solution's of x2-4=0 To promote accountability, which document outlines the method for coordinating response operations?National Incident Management System (NIMS) DoctrineIncident Action PlanO Resource Tracking SystemO Responder Personnel RosterCheck Parents that are heterozygous for having freckles (dominant) are crossed.What represents the ratio of their offspring? Draw a Punnett square and show your work. Find the volume of the pyramid. Charles wants to analyze his last 9 math test scores with abox plot to see how spread apart they are.Arrange the scores in order, from lowest (on the left) tohighest 3.How do memories and childhood experiences shape a person's identity later in life?Describe a memory from your childhood that you consider important to who you are today. In the U.S., the media uses results of exit polls to predict the outcome of elections. These exit polls ask people who just voted during election day questions such as who they voted for and why did they vote for them. A number of countries have outlawed exit polls because they believe that exit polls can affect the outcomes of elections. For example, voting precincts in the West Coast (e.g., California, Washington, Oregon) close later than those in the East Coast (e.g., Virginia, New York, New Jersey). Because of this fact, people in the West Coast may end up deciding not to vote because the exit poll results already predicted a winner. In this case, people may feel like their votes will not matter at all, so they don't end up casting their votes. As such, there is a growing desire among people in the U.S. to restrict the use of exit polls. Which of the following statements, if true, would most strengthen the above argument?1. The media has stopped announcing results of exit polls until all voting precincts are closed.2. Countries that have outlawed exit polls have higher voting participation rates than countries that allow exit polls.3. Exit poll results have proven to be accurate predictors of election results. Graph the equation y = 1/3x - 5 In which country was Sidhartha Guayana born? Two sides of an obtuse triangle measure 10 inches and 15 inches. The length of longest side is unknown. What is the smallest possible whole-number length of the unknown side? 18 inches 19 inches 24 inches 25 inches Which multimedia aid provides the best support for thisclaim?a brief video clip of a medical school professordiscussing a specific case of using virtual realitya podcast about the origins of virtual reality technologyat medical schoolsa photo showing differences between virtual realityequipment for gamers and medical studentsa student discussion about the pros and cons of virtualreality in medicine Write a letter to the Editor in about 120-150 words to the Times of India' regarding the frequent traffic jams on the roads of your city because of poor road conditions. You are Priyank/ Priya Thomas, Mahatma Gandhi Rd, Andheri West, Mumbai. What is an aqueduct? Water channelSpecies of duckAncient structureWater parkJohn Smeaton built a lighthouse out of lime after previous engineers built them out of wood. What part of the engineering design process is this? *25 pointsIterateBrainstormIdentifyDefineWhich of the following is the first city with a working sewer system?25 pointsNYCLondonMilanBeijing Natasha wants to find out if the neighborhood supports lowering the speed limit on the street in front of her school. Which is a valid sample for the survey? students in Natashas social studies class parents who live in a town 15 miles away from Natashas school the first 5 drivers who drive past Natashas school one morning thirty residents who live within a 2-miles radius of Natashas school Some theorists have argued that modernity and post-modernity have brought with them a decline in communities. Do you agree or disagree? The distance covered by a car at a time, t is given by x = 20t + 6t4, calculate(i) the instantaneous velocity when t = 1 (ii) the instantaneous acceleration when t =1 Given the choice of 4 bases and the number of bases needed to make up a codon, how many possible codons are there? 16 23 64 128